Difference between revisions of "2014 AMC 12B Problems/Problem 3"

(Solution)
Line 9: Line 9:
 
If the first and last legs of his trip account for <math>\frac{1}{3}</math> and <math>\frac{1}{5}</math> of his trip, then the middle leg accounts for <math>1 - \frac{1}{3} - \frac{1}{5} = \frac{7}{15}</math>ths of his trip.  This is equal to <math>20</math> miles.  Letting the length of the entire trip equal <math>x</math>, we have  
 
If the first and last legs of his trip account for <math>\frac{1}{3}</math> and <math>\frac{1}{5}</math> of his trip, then the middle leg accounts for <math>1 - \frac{1}{3} - \frac{1}{5} = \frac{7}{15}</math>ths of his trip.  This is equal to <math>20</math> miles.  Letting the length of the entire trip equal <math>x</math>, we have  
 
<cmath>\frac{7}{15}x = 20 \implies x=\boxed{\textbf{(E)}\ \frac{300}{7}}</cmath>
 
<cmath>\frac{7}{15}x = 20 \implies x=\boxed{\textbf{(E)}\ \frac{300}{7}}</cmath>
 
(Solution by kevin38017)
 

Revision as of 22:34, 20 February 2014

Problem

Randy drove the first third of his trip on a gravel road, the next $20$ miles on pavement, and the remaining one-fifth on a dirt road. In miles, how long was Randy's trip?

$\textbf{(A)}\ 30\qquad\textbf{(B)}\ \frac{400}{11}\qquad\textbf{(C)}\ \frac{75}{2}\qquad\textbf{(D)}}\ 40\qquad\textbf{(E)}\ \frac{300}{7}$ (Error compiling LaTeX. Unknown error_msg)

Solution

If the first and last legs of his trip account for $\frac{1}{3}$ and $\frac{1}{5}$ of his trip, then the middle leg accounts for $1 - \frac{1}{3} - \frac{1}{5} = \frac{7}{15}$ths of his trip. This is equal to $20$ miles. Letting the length of the entire trip equal $x$, we have \[\frac{7}{15}x = 20 \implies x=\boxed{\textbf{(E)}\ \frac{300}{7}}\]